The number of chocolate chips in a bag of chocolate chip cookies is approximately normally distributed with a mean of 1262 chips and a standard deviation of 117 chips. ​(a) Determine the 28th percentile for the number of chocolate chips in a bag. ​(b) Determine the number of chocolate chips in a bag that make up the middle 97​% of bags. ​(c) What is the interquartile range of the number of chocolate chips in a bag of chocolate

Answers

Answer 1

Answer:

Using z score formula:

X = z ∂ + µ

 = 157.833

Step-by-step explanation:

Solution:

Mean = µ = 1262

Standard deviation = ∂ = 117

(a) 28th percentile for the number of chocolate chip.

P( z < z) = 28%

             = 0.28

P( z<- 0.58)  = 0.28

Z = -0.58

By using z score formula:

Z = x - µ /∂

-0.58= x – 117 / 1262

X = (- 0.58)(117) + (1262)

 =  1194.14

(b) Middle 97% of bag.

P(-z < z < z) = 97%

                    = 0.97

P( z < z) – p(z < -z) = 0.97

2p(z < z) -1 = 0.97

2p (z < z) = 1 + 0.97

P(z < z) = 1.97 / 2

            = 0.99

P(z < 2.33) = 0.99

Z ± 2.33

By using z score formula:

Z = x - µ / ∂

X = z ∂ + µ

  = - 2.33 x 117 + 1262

  =989.39

Z = 2.33

X = z ∂ + µ

  =  2.33 x 117 + 1262

 =1533.61

(c) What is the interquartile range of the number of chocolate chips in a bag of chocolate.

By using standard normal table,

The z dist’n formula:

P(z < z ) = 25%

            =0.25

P(z < -0.6745) = 0.25

Z = 0.6745

Using z score formula:

X = z∂ + µ

   = - 0.6745 x 117 + 1262

  = 1183.0835

First quartile = Q1 =1183.0835

The third quartile is:

P(z<z) = 75%

    = 0.75

P(z < 0.6745) = 0.75

Z = 0.6745

Using z score formula:

X = z ∂ + µ

  = 0.6745 x 117 + 1262

= 1340.9165

IQR = Q3 – Q1

     = 1340.9165 – 1183.0835

  = 157.833


Related Questions

what value is equivalent to 8. 9 - 2 . 5

Answers

6.4 is the answer. You just subtract

Answer:

6.4

Step-by-step explanation:

Hope this helps :)

Help please!!! Ty to anyone who answeres

Answers

Answer:

Scale factor=2

The perimeter of the enlarged figure is 19

The area of the enlarged figure is 4 times the area of the original figure

Step-by-step explanation:

Scaling

There are two figures, the red rectangle (original) and the blue rectangle (enlarged).

To find the scale factor we directly compare the height of both rectangles and note:

height of original rectangle=1.5

height of enlarged rectangle=3

Thus, the scale factor is 3/1.5 = 2

Scale factor=2

The original perimeter is calculated as:

P1=2*1.5+2*3.25=3+6.5=9.5

The perimeter of the enlarged figure is 2 times the original perimeter

P2=2*9.5=19

The perimeter of the enlarged figure is 19

To calculate the area of the enlarged figure, we use

A2=H2*W2

Where H2 and W2 are the height and width of the enlarged figure.

Since

H2=2*H1

W2=2*W1

A2=4*A1

The area of the enlarged figure is 4 times the area of the original figure

Please help me I don’t get this

Answers

Sum of the interior angles of any triangle equal 180° .

Thus ;

[tex]x + 2x + x + 8 = 180[/tex]

[tex]4x + 8 = 180[/tex]

Subtract sides -8

[tex]4x + 8 - 8 = 180 - 8[/tex]

[tex]4x = 172[/tex]

Divided sides by 4

[tex] \frac{4}{4}x = \frac{172}{4} \\ [/tex]

[tex]x = 43[/tex]

So the angles are :

x = 43°

2x = 2 ( 43 ) = 86°

x + 8 = 43 + 8 = 51°

Thus the order from least to greatest ,

is : 43° , 51° , and 86°

_________________________________

And we're done.....♥️♥️♥️♥️♥️

A salesperson is paid a flat rate plus a fixed percentage of her sales. Last month, she sold $16,000 worth of goods and was paid $1,600. Two months ago, she had $12,000 in sales and was paid $1,400. This month she sold $11,000 worth of goods. How much will she be paid?​

Answers

Answer:

She will be paid $1,350

Step-by-step explanation:

Linear Modeling

Some events can be modeled as linear functions. If we are in a situation where a linear model is suitable, then we need two sample points to make the model and predict unknown behaviors.

The linear function can be expressed in the slope-intercept format:

y=mx+b, where m and b are constants.

The payments for a salesperson will be linearly modeled. There are two known points: When the sales were $16,000, the payment was $1,600. This makes the point (16,000;1,600).

We also know when the sales were $12,000, the payment was $1,400. The point is (12,000;1,400)

Let's use the points to find the values of m and b.

Using (16,000;1,600):

1,600=m*16,000+b

Using (12,000;1,400):

1,400=m*12,000+b

Subtracting both equations:

200=16,000m-12,000m

200=4,000m

Solving:

m=200/4,000=0.05

Using the first equation and the value of m:

1,600=0.05*16,000+b

1,600=800+b

Solving:

b=800

The equation is now complete:

y=0.05x+800

She sold $11,000 this month, so the payment is:

[tex]y=0.05\cdot 11,000+800[/tex]

Y=550+800=1,350

She will be paid $1,350

Answer:

1350

Step-by-step explanation:

(Note: this method is not the best to use in future problems, please refer to the other answer if you want something better to use.)

16,000-12,000=4,000

1,600-1,400=200

To find the rate of change in 1000s:

200/4 = 50

Essentially this means the rise is 50 and the run is 1000, so, just remove $50 from the payment from 2 months ago to find your answer

PLEASE HELP WILL AWARD BRAINLIEST!! A company makes chairs, loungers, and footstools. Each item uses units of wood, fabric, and stuffing. Chairs use 30 units of wood, 30 units of fabric, and 10 units of stuffing. Loungers use 25 units of wood, 15 units of fabric, and 10 units of stuffing. Footstools use 20 units of wood, 5 units of fabric, and 5 units of stuffing. There are 1,380 units of wood, 890 units of fabric, and 450 units of stuffing available to make the items. Which three equations are needed to find the largest number of x chairs, y loungers, and z footstools that can be made? Drag and drop the correct equations into the box.

Answers

Answer:

30x+25y+20z=138030x+15y+5z=89010x+10y+5z=450

Step-by-step explanation:

in this problem we are expected to formulate the constraints

let chairs, loungers and footstools be x, y and z respectively

1. For wood we have

30x+25y+20z=1380

2 . For fabric we have

30x+15y+5z=890

. For stuffing we have

10x+10y+5z=450

therefore the three equation are

30x+25y+20z=1380

30x+15y+5z=890

10x+10y+5z=450

Anna found a piece of sediment with a diameter of 0.977 millimeters. Write the size of Anna’s sample in expanded form without exponents

Answers

Answer:

The answer in the expanded form is (0.9 + 0.07 + 0.007) millimeters

Step-by-step explanation:

Here, we want to write the size of the diameter of the sediment in the expanded form without exponents

Thus, we have;

0.977 = 0.9 + 0.07 + 0.007

The size of Anna's sample in expanded form can be written be expanding the decimal places.

The expanded form without exponents is (0.9 + 0.07 + 0.007).

Given:

The given diameter is 0.977 millimeters.

Separate the two decimal place of given number.

[tex]0.977=0.9+0.077[/tex]

Separate the three decimal place of given number.

[tex]0.977=0.9+0.07+0.007[/tex]

The expanded form without exponents is (0.9 + 0.07 + 0.007).

Learn more about number expansion here:

https://brainly.com/question/1665479

Tommy Flanagan was telling you what he ate yesterday afternoon. He tells you, "I had either popcorn or raisins. Also, if I had cucumber sandwiches, then I had soda. But I didn’t drink soda or tea." Of course you know that Tommy is the worlds worst liar, and everything he says is false. What did Tommy eat? Justify your answer by writing all of Tommy’s statements using sentence variables (P, Q, R, S, T), taking their negations, and using these to deduce what Tommy actually ate.

Answers

Answer:

tom ate sandwiches of cucumber and also had tea

Step-by-step explanation:

The computation is shown below:

P:I = popcorn

Q:I  = raisins

R:I = cucumber sandwiches

S:I = sodas

T:I = tea

1. It could be either popcorn or raisins

2. If i take sandwiches of cucumber so it also had soda

3. Do not drink soda or tea

Now considered the negation of 1, 2 and 3

So,

[tex]\sim (PVQ) = \sim P \wedge \sim Q\\\\\sim (R \to S) = \sim(\sim R\vee S) = R \wedge \sim S\\\\ \sim(\sim S \vee \sim T) = S \wedge T[/tex]

Now combining 4, 5 and 6

We get

[tex]\sim P \wedge \sim Q \wedge R \wedge \sim S \wedge S \wedge T\\\\\sim P \wedge \sim Q \wedge R \wedge T[/tex]

So, tom ate sandwiches of cucumber and also had tea

Carmelita used the multiplication table below to write ratios that are equivalent to StartFraction 3 Over 7 EndFraction. A multiplication table. In the row labeled 3, the numbers 3, 6, 9, 12, 15, 18, 21, 23, and 27 are highlighted. In the row labeled 7, the numbers 7, 14, 21, 28, 35, 42, 49, 56, and 63 are highlighted. One number is missing from the equivalent ratio, as shown. StartFraction 21 Over Blank EndFraction What is the missing number? 7 9 24 49

First correct answer gets brainlest.​

Answers

Answer:

The answer is 49.! :)

Step-by-step explanation:

If you look at the table, it shows 3;7. If you look at the 21 in the 2 row, and look under it in the other orange row its 49. So the answer would be 49! :)

Step-by-step explanation:

Answer:

49

Step-by-step explanation:

Cody has two bags of counters, bag A and bag B.Each of the counters has either an odd number or an even number written on it. There are 10 counters in bag A and 7 of these counters have an odd number written on them. There are 12 counters in bag B and 7 of these counters have an odd number written on them. Cody is going to take at random a counter from bag A and a counter from bag B.
a) Complete the tree diagram.
Harriet also has a bag of counters. Each of her counters also has either an odd number or an even number written on it. Harriet is going to take at random a counter from her bag of counters. The probability that the number on each of Cody's two counters and the number on Harriet's counter will all be even is 3/100.
b)Find the least number of counters that Harriet has in her bag.

Answers

monke monked mon key

Which equation has the solution n = 11?
7n - 8 = 62
8n + 6 = 102
3n + 12 = 45
5n - 20 = 25
URGENT!!!

Answers

Answer:

3n + 12 = 45

Step-by-step explanation:

3n + 12 = 45

3n = 45 - 12

3n = 33

n = 33

3

n = 11

Answer:

3n + 12 = 45

Step-by-step explanation:

*Process of elimination*

___________________

3n + 12 = 45

(given)

_________

3n + 12 = 45

- 12 - 12

(subtraction property of equality)

__________________________

3n = 33

÷ 3 ÷ 3

(division property of equality)

_______________________

n = 11

_____

7n - 8 = 62

(given)

________

7n - 8 = 62

+ 8 + 8

(addition property of equality)

__________________________

7n = 70

÷ 7 ÷ 7

(division property of equality)

_______________________

n = 10 ≠ 11

________

8n + 6 = 102

(given)

________

8n + 6 = 102

- 6 - 6

(subtraction property of equality)

__________________________

8n = 96

÷8 ÷8

(division property of equality)

_______________________

n = 12 ≠ 11

________

5n - 20 = 25

(given)

________

5n - 20 = 25

+ 20 + 20

(addition property of equality)

__________________________

5n = 45

÷5 ÷5

(division property of equality)

_______________________

n = 9 ≠ 11

help will do brainliest uwu I thought of a number, I added 1 3 4 to it, then I multiplied the result by 2 2 11 and I got 8. What was my number?

Answers

Answer:

152

Step-by-step explanation: 1+3=4+4=8x2=16x2=32x11=152

find the equation of the linear function represented by the table below in slope- intercept form. picture below of the table

Answers

The equation of the linear function is y = -4x + 1. You’re welcome

The speed of light is 3 x 10^8 meters per second. The sun is approximately 230.000.000 meters from Mars. How many seconds does it take for the sunlight to reach mars? Write your answer in scientific notation using the format: 4.2x10^5 With no spaces Round to the nearest tenth. Do not include units (Seconds) in your answer.
URGENT

Answers

Answer:

0.8

Step-by-step explanation:

Given that:

Speed of light = 3 * 10^8 m/s

Distance of sun to Mars = 230,000,000 meters

How many seconds does it take for sunlight to reach Mars?

Using the relation :

Speed = distance / time

Hence ;

Time = Distance / Speed

Time = 2.3 * 10^8 / 3 * 10^8

Time = 0.766

Time = 0.8 ( nearest tenth)

Employed Graduates: According to the administration at a particular college, 55% of graduates are typically employed full time after graduation. To assess the impact of the current job market, the administration will take a survey of 619 recent graduates. The administration finds that 52% of the surveyed recent graduates were employed full time. If the population proportion of all recent graduates employed is the same as the stated proportion typically employed, how likely is it that the administration would have obtained a sample proportion as low as or lower than it obtained from its survey

Answers

Answer:

The likelihood is    [tex]P(p <  \^ p) = 0.066807[/tex]

Step-by-step explanation:

From the question we are told that  

   The population proportion is p =0.55

    The sample size is  n  =  619  

   The sample proportion is  [tex]\^ p =  0.52[/tex]

Generally the mean of the sampling distribution is  [tex]\mu_{x} =  p = 0.55[/tex]

The standard deviation is  [tex]\sigma =  \sqrt{\frac{p(1- p )}{n} }[/tex]

=>    [tex]\sigma =  \sqrt{\frac{0.55(1- 0.55 )}{619} }[/tex]

=>     [tex]\sigma =  0.02[/tex]

Generally the likelihood that  the administration have obtained a sample proportion as low as or lower than it obtained from its survey is mathematically represented as

[tex]P(p <  \^ p) =  P(\frac{p- \mu}{\sigma }  < \frac{ 0.52- 0.55}{0.02 }   )[/tex]

Generally [tex]\frac{p- \mu}{\sigma }  =  Z(The \  standardized \  value  \  of  \  p)[/tex]

=>   [tex]P(p <  \^ p) =  P(Z <-1.5  )[/tex]

From the z-table the probability of (Z <-1.5  ) is  

       [tex]P(Z <-1.5  )  = 0.066807[/tex]

So

      [tex]P(p <  \^ p) = 0.066807[/tex]

 

Help me please is the math

Answers

Answer:

2nd) which is 4 multiplied by 5 multiplied by 2

Question
An amount of $290,000 is borrowed for a period of 25 years at an interest rate of 4%. The amortization schedule for this
loan is below. Payments of $1,530.73 are made monthly.
Payment #
1
2
Payment Interest
1,530.73 966.67
1, 530.73964.79
1.530.73 962.90
Debt Payment
564.06
565.94
567.83
Balance
289, 435.94
288, 870.00
288, 302.17
3
4
X
5
Calculate x,the balance on the loan at the end of month 4. Give your answer to the nearest dollar. Do not include commas
or the dollar sign in your answer.
Provide your answer below:

Answers

The Answer is

$287732

I wish you all the best of luck

jenns age is 3 years less than marks age the sum of their ages is 30 let m be marks age find Jenna age

Answers

Answer:

you could do 27+x=30

Step-by-step explanation:

A segment has endpoints A (-1, 1) and B (8, 4) .
If the segment is divided into four equal parts, the coordinates of the point closest to point A are _____?

Answers

Answer:

fafafsghdjdbdbdgsysvdve. wwmsk

find the equation of the line that has the given slope and passes through the given point. write the equation in the slope intercept form. m=2 (5,4)

Answers

Answer:

y = 2x - 6

Step-by-step explanation:

Use the slope-intercept form of the equation of a line.

y = mx + b

y = 2x + b

4 = 2(5) + b

4 = 10 + b

b = -6

y = 2x - 6

The prostate-specific antigen (PSA) test is a simple blood test to screen for prostate cancer. It has been used in men over 50 as a routine part of a physical exam, with levels above 4 ng/mL indicating possible prostate cancer. The test result is not always correct, sometimes indicating prostate cancer when it is not present and often missing prostate cancer that is present. Suppose that these are the approximate conditional probabilities of a positive (above 4 ng/ml) and negative test result given cancer is present or absent.
Test Result
Positive Negative
Cancer present 0.21 0.79
Cancer absent 0.06 0.94
In a large study of prostate cancer screening, it was found that about 6.5% of the population has prostate cancer.
A. Draw a tree diagram for selecting a person from this population (outcomes: cancer present or absent) and testing his blood (outcomes: test positive or negative).
B. What is the probability that the test is positive for a randomly chosen person from this population?

Answers

Answer:

Step-by-step explanation:

A tree diagram is the right way of determining a list of sample elements in the sample space. It is by finding the ordered n-tuples of the elements in the events under consideration.

We can found the tree diagram for selecting a person from the population in the given question in the attached image below.

b. Given that 6.5% of the population have prostate cancer. Suppose the total population is 100000. Then, we have the following table:

By applying Naive-Bayes theorem, the probability that the test is positive for a randomly chosen person from this population is expressed as:

P( positive ) = P( positive | cancer present ) P( cancer present ) + P( positive | cancer absent ) P( cancer absent )

P( positive ) = 0.21×0.065 + 0.06×(1 - 0.065)

P( positive ) = 0.01365  + 0.0561

P( positive ) = 0.06975

Thus, the probability that the test is positive for a randomly chosen person from this population is:

= 0.06975

Shania is taking a survey about her attitude towards children’s programming. Even though she still watches Sesame Street every night before she goes to bad, she is afraid she will be judged so she answers dishonestly. This is an example of which problem of surveys?

Answers

Answer:

Response Bias

Step-by-step explanation:

This is an example of the problem known as Response Bias. This is basically any and all reasons an individual may have for not answering a question truthfully regardless of the reason. This is a huge problem when conducting a survey since there are almost no ways to prevent this and false responses can completely contaminate the results gathered by the survey. In this scenario, the reason for lying was fear and shame which are normal emotions for individuals and is impossible to predict who will lie due because of it.

What integer represents you getting $200 from the bank

Answers

Answer:

positive 200 or +200

Step-by-step explanation:

QUESTION 05 (10 points) - Sample Size Determination in Estimating Population Mean In this hypothetical scenario, you are working for a consumer protection agency. Your boss assigned you the task of conducting a survey on cheese burgers at a national restaurant chain. In your report, you would need to construct a 90% confidence interval for the mean weight of cheese burgers. Based on past surveys on cheese burgers at this restaurant chain, the standard deviation of weight is 0.21 pound. It is required that the maximum likely sampling error of mean weight be 0.05 pound. A cheese burger at the restaurant chain costs $1.29. What is the required budget in dollars (2 decimals) to have the cheese burgers in your sample, assuming no sales tax

Answers

Answer:

$ 37.32 is the required budget in dollars (2 decimals) to have the cheese burgers in your sample

Step-by-step explanation:

Here

Z∝ =1.64

Standard Deviation=S= 0.21

Sampling Error= SE= 0.05

As Sampling Error is found by

SE= z * s/√n

√n= z*s/ SE

n= (z* s/ SE)²

We use the above formula  to calculate N

N= ( Z * S/SE)²

N= (1.64 * 0.21/0.05)²

N= 28.9296= 28.93

Cost of N number of burgers

= Cost *N

= $1.29 * 28.93= $ 37.32 is the required budget in dollars (2 decimals) to have the cheese burgers in your sample

Write the word sentence as an equation. Then solve.
The sum of a number y and -3 is –8.
Equation:
Solution: y =
24

Answers

Answer:

The equation is: y + -3 = 8

And y = -5

Step-by-step explanation:

Sum means addition which means the sum of a number y and -3 is the same as y + -3. So, you would add the =8's to make it an equation.

Use the drop down menus to complete the statement to match the information shown by the graph

Answers

I GOT YALLLLLL LIKE I RISK MY GRADE TO HELP YALL

Hasan travels

at the same speed as

Anneliese because the

slope

of Hasan's graph is

equal to

that of Anneliese's graph.

The

slope

of the graph tells you the unit rate is in

meters per minuters

-5x – 4y = -8
Write a formula for f(x) in terms of x

Answers

5x+4y-8= 0

Step-by-step explanation:

Move the constant to the left change its sign

5x-4y=-8 look at the terms of x

what is 4/11 divided by 4/9 as a fraction

Answers

Answer:

9/11

Step-by-step explanation:

To divide, you have to flip the dividend.

So, 4/9 would become 9/4

Then you multiply 4/11 by 9/4

This would give you 36/44

Simplify.

18/22

Simplify some more.

9/11

Hope I helped :)

Please consider Brainliest :)

Which transformation reflects f(x)=(x-1)² over the y-axis?

A) f'(x) = 0.3((x-1)²+1)

B) f'(x) = -3((x-1)²+1)

C) f'(x) = (-4x-1)²+1

D) f'(x) = (0.1x-1)²+1​

Answers

Answer:

A) f'(x) = 0.3((x-1)²+1)

Step-by-step explanation:

PLZ ANSWER FAST (40 POINTS + BRAINLIEST)

Rewrite the expression [tex]y\frac{2}{3}[/tex] as a radical expressions.

Answers

Answer:

[tex] \sqrt[3]{y {}^{2} } [/tex]

What is the answer to 1/6 plus 2/3

Answers

Answer:

5/6

Step-by-step explanation:

First, you need to make sure they have a common denominator, or else addition is not possible.

Luckily, in this case, 6 and 3 are divisible.

You would multiply 2 by 2/3 to get the common denominator of 6

The new expression would be:

4/6 + 1/6

Now, you just need to add the numerators

4 + 1 = 5

The correct answer would be 5/6.

Hope this helps! Good luck :)

Answer: 5/6

do you have a calculator or sum, cuz it's pretty easy

Other Questions
A moving truck rental company charges $26.95 to rent a truck, plus $0.8 per mile. Suppose the function C(d)gives the total cost of renting the truck for one day if you drive 40 miles.Give the formula for C(d). Make sure to give the complete formula as an equation.CorrectGive the total rental cost if you drive the truck 40 miles. Give the function notation in the first box, the answer in the second box, and choose the correct units from the third box.Correct ASAP Which statement best describes the governments of the city-states?The city-states voted on which city-state would lead the rest.Each city-state was independent and had its own form of government.The city-states were all part of one empire with the same government.All city-states had a similar form of government, since they were located near one another. I want to know the answer to this PLEASE HELPPP The American Revolution was considered one of the causes of the French Revolution because A: it showed the French people it was possible to overthrow a King and powerful army B: the French people that Americans could kill a king C: It was possible for poor people to become King D: it was impossible to defeat a king with a strong army and navy The idea that even if something is out of sight, it still exists is called ________. You start at (6,7) and move 5 units right and 4 units up.Plz help me A ball is thrown vertically upward with an initial velocity of 29.4 m/s. What is the maximum height reached by the ball? How long is the ball in the air? What is the final velocity of the ball before it goes back to its original position? 7. Which of the following is created when anenclosed fluid is squeezed?A. buoyancyB. convectionstC. pressureD. gravity Someone who was given a land grant by Mexico to settle an area of Texas and was responsible for settlers on thatland was called a(n)a baronb colonistC governord. empresarioPlease select the best answer from the choices providedAB explain how the removal of the species changes the biodiversity in the ecosystem Volunteers at Sams school use some of the student councils savings for a special project.They buy 4 backpacks for $9 each and fill each backpack with paper and pens that cost $5.By how much did the student councils savings change because of this project? What is the difference between em waves and mechanical waves? reply with anything :D When the air resistance can be ignored the velocity of an object dropped initially from rest is given by following equation where g is free fall accelerationg*t^2/2g*tg*t/2g Can somebody plz help? why do our digestive systems produce separate enzymes to digest proteins, fats, and carbohydrates? Given the perimeter of a rectangle and either the length or width, find the unknown measurement using the appropriate formula.The perimeter is 70ft . and the width is 16ft . John had $800 Tasha has $500 Kyle had $300 Who had the most money? The operations of Winston Corporation are divided into the Blink Division and the Blur Division. Projections for the next year are as follows: Blink Division Blur Division Total Sales $ 380,000 $ 198,000 $ 578,000 Variable costs 118,000 97,000 215,000 Contribution margin $ 262,000 $ 101,000 $ 363,000 Direct fixed costs 104,000 90,000 194,000 Segment margin $ 158,000 $ 11,000 $ 169,000 Allocated common costs 59,000 51,500 110,500 Operating income (loss) $ 99,000 $ (40,500 ) $ 58,500 If the Blur Division were dropped, Blink Division's sales would increase by 30%. If this happened, the operating income for Winston Corporation as a whole would be: If [tex] \cos(330) = \frac{ \sqrt{3} }{2} [/tex]Prove that:[tex] \tan(165) = \sqrt{3} - 2[/tex]Please solve this question.